Let's use this discussion to brainstorm about possible interesting topics that you would like to write about / learn more about.
I have a lot of confidence in the members of this group, and believe that you can help create a fun interesting experience for other Brilliant members. I'd check in occasionally, so please send me an email if you need an urgent response.
Here are some suggestions:
1) Michael Tong - olympiad style problems
2) Sotiri - problems based on what we do at math circle every other Saturday
3) Anqi - Elementary techniques used in IMO
4) Nina - mathematical stories/resources/interesting tidbits - I have lots of reading suggestions, youtube channel suggestions, and random amazing facts about the beauty of mathematics!
5) Piyushkumar - Algebra, Geometry, Number Theory to clear RMO, INMO, IMO
Easy Math Editor
This discussion board is a place to discuss our Daily Challenges and the math and science related to those challenges. Explanations are more than just a solution — they should explain the steps and thinking strategies that you used to obtain the solution. Comments should further the discussion of math and science.
When posting on Brilliant:
*italics*
or_italics_
**bold**
or__bold__
paragraph 1
paragraph 2
[example link](https://brilliant.org)
> This is a quote
\(
...\)
or\[
...\]
to ensure proper formatting.2 \times 3
2^{34}
a_{i-1}
\frac{2}{3}
\sqrt{2}
\sum_{i=1}^3
\sin \theta
\boxed{123}
Comments
So here's a rough draft of what I might post, and notice that it would probably be less of just problems, but explanations first, then practice problems for the interested reader. These will be like excursions into IMO standard problems for the novice. So please give your comments on how this can be improved :)
Invariants
The Principle
Note: I assume the reader has familiarised himself with Introduction to Invariance Principle
Invariants are particularly useful in analysis of games, transformations and questions of the like. Basically, the main motivation behind applying the invariance principle is to find something that does not change, when something is repeated, whence the important usage of this in algorithms. Quoting Problem-Solving Strategies, here is a non-exhausted list to keep in mind when applying invariance.
Can a given state be reached?
Find all reachable end states.
Is there convergence to an end state?
Find all periods, if possible.
Also, if you are ambitious and want to use this in olympiad combinatorics problems, here's a few common invariants that we will encounter along the way:
Parity
Colouring (as in the more commonly known checkerboard colouring and others)
Sum of squares/products or things of the like
Now, also to introduce another method in invariance, we will start with our first example:
Example 0 : 23 friends wants to play hockey. However, to ensure that the game is fair play, they choose a referee and the rest of the team splits into 2 teams of 11 people each. However, because they are a bit eccentric, they want to do this splitting such that the total weight of each team is the same. Suppose now for simplicity sake that they all have integer weights, and we know that this division is always possible regardless of who is the referee. Show that they all have the same weight.
Solution: Wow, that is a really really long question and to help us understand the context and meaning, we rewrite this in mathematical equations. Let w1,w2,…,w23 be the weights of the 23 people. Also, let S=w1+w2+…+w23, the motivation behind this definition is that choosing a referee is the same as removing a person with weight wi and split the rest into 2 teams of weight k each. Now, we can write everything in this compact form: S−wi=2k(∗). Remember what we said about parity? Clearly since the RHS is even, wi and S needs to have the same parity. And since wi was completely arbitrary, they are all odd or even.
Being odd or even only gives us one clue on how to proceed, which is whether 2 divides it or not. Because we want to work with simplified things, consider what happens if we divide those that are even by 2. For instance, if all wi are even, if we let qi=2wi, does this simplify the problem to some extent? Well, clearly by definition all qi are integers, and writing a equation, 2S−qi=22k which is unmistakably (∗) so qi is a new list of weights that also satisfy the conditions. Analogously, we can conclude for wi being odd (don't forget this case!), we can let qi=wi−1.
Here's the important bit. If the wi at the beginning were not all 0, then the sum of the weights qi is strictly smaller. We can always repeat this step (this is sort of an algorithm-like approach - another method that works like induction; you use it if you do not have any idea how to proceed) and reduce the sum of the weights so eventually we will reach a list of only zeroes. Now we backtrack to see what this implies. So since we are able to do this, we conclude the numbers in the original list were all equal, as desired. □
So here, instead of finding something that is not changing, we in fact created something to make S always decreasing. The main idea here is that there is no infinite sequence of non-negative integers that is strictly decreasing. This technique is more commonly known as infinite descent. We will encounter a much more difficult application of this later.
Applications
In these examples, we will go into detail the motivation and thought process behind each key step. The last few questions are taken from the IMO, so expect that it will be much more convoluted and difficult to approach than the rest.
[blah blah - I will write later when I finish my breakfast]
EDIT: So we proceed. The first example is of a game. We will see how to apply the invariance principle.
Example 1: On a chessboard A and B play by turns to place black and white knights, respectively. Either player loses when he places a knight on a square attacked by a knight of the other colour or there are no free squares to place the knight on. If A starts, who has a winning strategy?
Solution: For those chess noobs like me, remark that a knight can only attack the 8 squares shown in the figure below:
tagtext
Intuitively, if each player can place a knight such that the squares that can be attacked by both knights do not overlap, then since a chessboard has an even number of cells, Player B can win, by in what people call, mirroring the first player.
Clearly a knight on a black square can only attack a knight on a white square and vice versa. So the colours of the squares that the knight attacks is invariant. Recall that a chessboard is a 8×8, so if we "fold" the board along the middle, the 2 "flaps" would be of opposing colour. For instance, let ai,j be the colour of the cell in the ith row and jth column of the chessboard. Then we have that a4,2=a4,7.
The above observation is key for us to see that the strategy for B is to do the same thing as A but symmetric to the center of the board. Basically, notice that if A can place a knight without being attacked by B's knights, the B can always place a "symmetric" knight without being attacked by A's knight due to the colour argument. Now given the way that knights attack, A can never place a knight that attacks the square where B played. Thus A loses in the end. □
To further train the skills of the reader in solving game-like problems (because we would soon be moving to colouring and other invariance, and I would select the IMO problems for demonstration), we will attempt one more example, this problem was in my training notes, and according to our trainer, was from Bulgaria 2001.
Example 2 Alice and Bob play by turns to write ones and zeroes in a list, from left to right. The game ends when each has written 2001 numbers. When the game ends the sequence of 0,1 is interpreted in base 2. Alice wins if that number can be written as a sum of two perfect squares and if otherwise, Bob wins. Who has a winning strategy?
Solution: We want a invariant that somehow involves squares. Well, preliminarily, just plain squares would remind me of (mod4). So ultimately one should think of Fermat's Christmas Theorem. Let's see how this property translates to a winning strategy.
Now suppose the number is 1100…0 where there is an even number of 0, what can you observe? (Hint: consider (mod4)) Yup, exactly! Since 4m can be written as a sum of 2 squares iff m can, we can perfectly well ignore the zeroes at the end. And notice that 11 in base 2 cannot be written as a sum of 2 squares, so B wins! Generalising this argument, at any moment A writes a 1, B just copies A′s move. In this way, if we ignore the zeores, the final number would end in 11 which is congruent to 3(mod4) which by Fermat's Christmas Theorem, cannot be written as a sum of 2 squares, so B wins.
So remember that A is smart. She wants to avoid the above case where she loses. So she says: "Aha! I only write 0s, in that way, I will win!" Well, sad life A, because you don't. Because if A only writes "0s", using the same argument above we can in fact make B write 1998 times 0 and then 3 times 1, which after interpreting in binary is 21 so B wins again. □
[Will continue after urgent meeting]
EDIT again Let us now apply infinite descent to a much more difficult problem.
Example 3: (IMO 1986) We assign an integer to each vertex of a regular pentagon, so that the sum of all of them is positive. We are allowed to perform a transformation if three consecutive vertices have assigned numbers x,y,z respectively and y<0, if so, then we can change the numbers (x,y,z) to (x+y,−y,z+y). This process can be continued as long as one of the numbers is negative. Does this process always come to an end?
Solution: Don't freak out that this is from IMO.
tagtext
We label the pentagon with a,b,c,d,e as shown.
So we need to find a certain invariance. In fact the only "basic" thing we have is the sum of all numbers. Also notice that (x+y)+(−y)+(z+y)=(x+y+z) so the sum remains constant. This does not tell us whether the process ends. We need something that decreases. Now from the sum, it is certainly quite motivated to consider the average. So if every number is very close to the average then none would be negative and the process would end. So we want to search for something that will increase as the numbers are different to the average. Naturally, we might think of ∣x−y∣ but the absolute value can be quite unpredictable at times, so we consider the "nicer" alternative squares (x−y)2.
So referring back to the notation at the beginning, consider a function f(a,b,c,d,e)=(a−c)2+(c−e)2+(e−b)2+(b−d)2+(d−a)2. Clearly f(a,b,c,d,e)≥0. Suppose WLOG that c<0 and we do the transformation in the problem. Then:
f(a,b+c,−c,d+c,e)=(a+c)2+(−c−e)2+(e−b−c)2+(b−d)2+(d+c−a)2
=[(a−c)2+4ac]+[(c−e)2+4ce]+[(e−b)2−2ec+2bc+c2]+(b−d)2+[(d−a)2−ac+2cd+c2]
=f(a,b,c,d,e)+2c(a+b+c+d+e)
Since c<0 and we assumed that a+b+c+d+e>0, we have that 2c(a+b+c+d+e)<0 so f(a,b,c,d,e) decreases at each step. By infinite descent we are done. □
One last example that uses colouring.
Example 4: (IMO 2004) Define a hook to be a figure made up of 6 unit squares as shown in the diagram
tagtext
or any of the tiles by applying rotations and reflections to the diagram. Determine all m×n rectangles that can covered be hooks such that:
the rectangle is covered with hooks and no overlaps
no part of the hook covers an area outside the rectangle.
Solution: We hate hooks because they are in a way disjoint. We like tiles that are nice 2×3 rectangles or things of the sort. So it is pretty motivated to consider pairs of hooks due to the hole, as marked out (with a black dot) in the following diagram:
tagtext
So necessarily if tile X cover tile Y′s "hole", then necessarily tile Y also cover the "hole" of tile X. We have the following 2 pairs:
tagtext
We have our first breakthrough for this problem: 12∣mn. Consider a 2×6 board. Clearly we cannot tile it with the pairs of tiles. However, we can obviously tile a 4×6 board. We might now conjecture that we must have (WLOG) that 4∣m.
Let us consider the case where they are both even but 4∤m,n. So we want to find a colouring that makes use of the divisibility by 4 condition. Well, the most direct way of incorporating it this is to add a 1 to each cell of the board that belongs to a column or row divisible by 4. Basically what we want to do next can then be simplified to proving that each of the pairs of hook only cover an odd number of 1. (Do you see why this is sufficient?). Anyways, here's the board with the labelling:
tagtext
So each pair of hook clearly can only cover an odd number of 1 by doing a simple case-bashing (I leave the reader to convince himself). So there must be an even number of tile → 4∣m (WLOG).
Now, let us summarise what we have:
12∣mn
4∣m
m,n=1,2,5 (do you see why?)
(i) So if 3∣n then we can clearly tile the whole board using the 3×4 rectangles formed from the pair of hooks.
(ii) If 12∣m and by point 3, m≥6 then by a simple induction we can show that m can be represented in the form 3a+4b. The motivation behind this step is that we want to be able to split up the board into smaller sections of case (i) above, because intuitively the 3×4 rectangles is more predictable than the second kind of pairing. Then all we do is that we split up the board into a strips of 3×m and b strips of 4×m which both can be tiled. □
Example 5: (IMO shortlist 1998) A solitaire game is played on an m×n regular board, using mn markers that are white on one side and black on the other. Initially, each square of the board contains a marker with its white side up, except for one corner square, which contains a marker with its black side up. In each move, one may take away one marker with its black side up but must then turn over all markers that are in squares having an edge in common with the square of the removed marker. Determine all pairs (m,n) of positive integers such that all markers can be removed from the board.
Solution: This is a particularly annoying problem. It is extremely long and difficult to understand. As such, let us employ the technique of trying small cases to perhaps find a pattern.
m,n=1. This is boring. Just remove the "corner square" marker.
WLOG m=2,n=1. This is boring too. Remove any "corner square" marker. The other will be black. Remove both.
m=2,n=2. Just do the operations and you will find that you end up with a sole white marker that cannot be removed.
m=2,n=3. Just remove the first row one by one and leave the following scenario, it is clear that all markers can be removed.
m=2,n=4. Playing around shows that this case is impossible.
Diagram 1
These observations motivate us to conjecture that if either m,n is odd then we can remove all markers (using the 4th case as a hint). WLOG, let us write m=2k+1. For easy reference, we assign the markers a position (i,j) as the ith row and jth column. So now, again, WLOG, (1,1) is that with the black side up. We remove (1,1) and in sequential from top to bottom, remove all (k,1) for 2≤k≤m. We have something as in the following diagram:
tagtext
So in our mathematical language, each marker (k,2) now has its black side up (sounds like backside up). Anyways, the next thing is to sequentially (which means in the order of q=1,…,2m−1 ) remove the markers (2q+1,2). The cute thing here is that each with (2q,2) is actually flipped twice, which means it turn from black→white→black, which is nice. So we have the following diagram:
tagtext
Now by removing the markers (2q,2) sequentially, we end up with the following diagram. It is clear now that by using this procedure we can remove all markers given that m is odd (so that the final row would be black).
tagtext
To show that if (m,n) are both even then we cannot remove all the markers, we need to find an invariance. Here are 2 approaches, as we will see, the first approach stems from the fact that empty squares without markers are useless, so the idea of removing empty squares. The second one is more well-known, as assigning weights of −1,1 allows one to consider sums and products (as in this case).
Consider the perimeter of the board. We consider removing a black marker as removing the edges that the cell it is in shares with the perimeter of the board. This is a bit ambiguous and requires the formal definition of perimeter. With that in mind, we will refer to removing a black marker as removing the cell it is in. Consider removing a cell that shares k edges with the perimeter of the board, then k is lost and 4−k is gained for the perimeter. The presence of 4 suggests taking (mod4) which is a considerably good strategy. So, we have a net gain of 4−2k for the perimeter, which is 2k(mod4).
We want to eliminate the variable k, so let's give it a physical meaning first. Well, except for the first black marker and its cell, k is actually # of adjacent markers removed+# of edges shared with the perimeter of the board. Notice that the former is always odd, since if we consider the cycle white→black→…→Black, we would always need an odd number of manipulations/arrows, as desired. This implies that the total sum is odd, and we see that no matter whether k≡1,3(mod4), 2k≡2(mod4).
However, this means the perimeter changes (n−2)(m−2)+3 times from 2(m+n) which is the original perimeter. By assumption 2(m+n)≡0(mod4), which means that the final perimeter is 2(mod4). It is clear that a contradiction arises since we need it to be 0(mod4). □
Example 6: (IMO shortlist 2005) There are n markers, each with one side white and the other side black, aligned in a row with their white sides up. At each step, if possible, we choose a marker with the white side up (but not one of the outermost markers), remove i, and reverse the two neighbouring markers. Prove that one can reach a configuration with only two markers left if and only if 3∤n−1.
Solution: Just playing around with small examples can lead to one trivial observation: the parity of the number of black markers remains unchanged during the game. Hence if only two markers are left, they must have the same colour.
As with what was written in Example 5, we can see that we can assign "weights". Well, here the best thing to encode would be the number of black markers. Remember that (−1)n for some n provides lots of information by encoding the parity(when we consider sums), let us define assign to a white marker with n black markers to its left the value (−1)n. We only assign values to the white markers. Denote S as the sum of all numbers assigned to the white markers.
Consider an arbitrary white marker with both of its neighbours initially white and with k black markers to its left. We perform an operation on that marker and see how it affects (−1)n. Clearly, if we turn both of its neighbouring markers black and remove it, then S decreases by −(−1)k+(−1)k−1+(−1)k−1=3(−1)k−1. Therefore, we have just shown that S(mod3) is invariant.
If the game ends with two black markers then S≡0(mod3) and if it ends with two white markers then S≡2(mod3). →3∤n−1.
To show that if we start with n≥5 markers and n≡0,2(mod3) satisfies the conditions. By removing the 3 leftmost white markers (that do not violate the conditions), we obtain a row of n−3 markers. By working backwards, we can reach 2 or 3 white markers. We are done with the former and for the latter we can reach 2 black markers. □
Practice Problems
Five identical empty buckets of 2-liter capacity stand at the vertices of a regular pentagon. Cinderella and her wicked Stepmother go through a sequence of rounds: At the beginning of every round, the Stepmother takes one liter of water from the nearby river and distributes it arbitrarily over the five buckets. Then Cinderella chooses a pair of neighbouring buckets, empties them to the river and puts them back. Then the next round begins. The Stepmother goal's is to make one of these buckets overflow. Cinderella's goal is to prevent this. Can the wicked Stepmother enforce a bucket overflow?
Let S be a finite set of at least two points in the plane. Assume that no three points of S are collinear. A windmill is a process that starts with a line ℓ going through a single point P \in S The line rotates clockwise about the pivot P until the first time that the line meets some other point belonging to S. This point, Q, takes over as the new pivot, and the line now rotates clockwise about Q, until it next meets a point of S. This process continues indefinitely. Show that we can choose a point P in S and a line ℓ going through P such that the resulting windmill uses each point of S as a pivot infinitely many times.
At the vertices of a regular hexagon are written six nonnegative integers whose sum is 20032003. Bert is allowed to make moves of the following form: he may pick a vertex and replace the number written there by the absolute value of the difference between the numbers written at the two neighboring vertices. Prove that Bert can make a sequence of moves, after which the number 0 appears at all six vertices.
(Optional) (a) Find a monovariant for Example 3 using absolute values.
(b) Solve Example 5 by assigning the weights as suggested as a second approach.
Hints:
Well, since we cannot spam this directly, consider working backwards. Clearly, Cinderella needs to ensure that the buckets have a1,…,a5≤1 or else the bucket would overflow on Stepmother's turn. Suppose then that the stepmother adds w1,…,w5 to the buckets, where w1+⋯+w5=1. So if we think about it further, we can see that if there are 2 nonadjacent buckets (we are trying to make the stepmother win now), the stepmother wins if we have ai+wi>1 and ai+2+wi+2>1. Cinderella needs to prevent the stepmother from setting (wi,wi+2)=(1+ϵ−ai,1+ϵ−ai+2), so all she does is to ensure that 2+2ϵ−ai−ai+2>1=w1+⋯+w5 for all ϵ>0→ai+ai+2≤1. Now it is just your job to prove that these invariants hold at every step.
There isn't a really detailed hint for this question. Remark first that: If we start with a point on the convex hull of S and a line ℓ that is “tangent” to the convex hull then we will only iterate over the points from the convex hull. The key motivation behind the solution is the fact that in rotations there are some really important angles such as π,2π. Also think, if we draw a line ℓ, then the plane is split into two sections and there is one point only on ℓ, how do we know that we can iterate? Maybe we can see what happens if we swap the two sections? Do you see the invariance?
Clearly we need to construct a monovariant. Think: Is 20032003 purposely there to put you off? Try some small cases. It becomes apparent that we need an alternating odd even structure. Prove that this is possible. Try casework if everything else fails. We need a monovariance that looks something like: max{a,b,c}−g(a,b,c) Here we define g(a,b,c)=# of a,b,c that are equal to max{a,b,c}. The alternating structure suggests strongly to use 3max{a,b,c}−g(a,b,c). Use induction to prove that this invariance holds a every step.
(a) Consider including pairwise sums then testing and etc.
(b) Refer to Example 6.
Sources: IMO problems and my SIMO senior training notes.
Note : Practice Problem 1 is from IMO 2009 Shortlist Problem C5 and Practice Problem 2 is from IMO 2011 Problem 2.
Some more invariance questions:
One: Is it possible to move a knight on a 5×5 chessboard so that it returns to its original position after having visited each square of the board exactly once?
Two: Prove that if we remove two opposite unit square corners from the usual 8×8 chessboard, the remaining part cannot be covered with 2×1 dominoes.
Three: On every square of a 1997×1997 board is written either +1 or −1. For every row we compute the product Ri of all numbers written in that row, and for every column we compute the product Ci of all numbers written in that column. Prove that ∑i=11997(Ri+Ci) is never equal to zero.
Four: There is one stone at each vertex of a square. We are allowed to change the number of stones according to the following rule: We may take away any number of stones from one vertex and add twice as many stones to the pile at one of the adjacent vertices. Is it possible to get 1989, 1988, 1990, and 1989 stones at consecutive vertices after a finite number of moves?
Five: Given a circle of n lights, exactly one of which is initially on, it is permitted to change the state of a bulb, provided that one also changes the state of every dth bulb after it (where d is a divisor of n strictly less than n), provided that before the move, all these n/d bulbs were in the same state as another. For what values of n is it possible to turn all the bulbs on by making a sequence of moves of this kind?
Six: Given a stack of 2n+1 cards, we can perform the following two operations: (a) Put the first k at the end, for any k. (b) Put the first n in order in the spaces between the other n+1. Prove that we have exactly 2n(2n+1) distinct configurations.
Seven: Three piles of stones are given. Sisyphus carries the stones one by one from one pile to another. For each transfer of a stone, he receives from Zeus a number of coins equal to the number of stones from the pile from which the stone is drawn minus the number of stones in the recipient pile (with the stone Sisyphus just carried not counted). If this number is negative, Sisyphus pays back the corresponding amount (the generous Zeus allows him to pay later if he is broke). At some point, all stones have been returned to their piles. What is the maximum possible income for Sisyphus at this moment?
Eight: In the sequence 1,0,1,0,1,0,3,5,0,…, each term starting with the seventh is equal to the last digit of the sum of the preceding six terms. Prove that this sequence does not contain six consecutive terms equal to 0,1,0,1,0,1, respectively.
Nine: At a round table are 1994 girls playing a game with a deck of n cards. Initially, one girl holds all the cards. At each turn, if at least one girl holds at least two cards, one of these girls must pass a card to each of her two neighbors. The game ends when and only when each girl is holding at most one card. (a) Prove that if n≥1994, then the game cannot end. (b) Prove that if n<1994 , then the game must end.
Ten: A solitaire game is played on an m×n regular board, using mn markers that are white on one side and black on the other. Initially, each square of the board contains a marker with its white side up, except for one corner square, which contains a marker with its black side up. In each move, one may take away one marker with its black side up but must then turn over all markers that are in squares having an edge in common with the square of the removed marker. Determine all pairs (m,n) of positive integers such that all markers can be removed from the board.
Eleven: Three numbers are written on a blackboard. We can choose one of them, say c, and replace it by 2a+2b−c, where a and b are the other two numbers. Can we reach the triple 11,12,13 from the triple 20,21,24?
Twelve: The sides of a polygon are colored in three colors: red, yellow, and blue. Initially, their colors read in the clockwise direction: red, blue, red, blue, ... , red, blue, yellow. We are allowed to change the color of a side, but we have to ensure that no two adjacent sides are colored by the same color. Is it possible that after some operations, the colors of the sides are in the clockwise order: red, blue, red, blue, ... , red, yellow, blue?
Thirteen: The numbers 1,2,…,100 are arranged in the squares of an 10×10 table in the following way: the numbers 1,…,10 are in the bottom row in increasing order, numbers 11,…,20 are in the next row in increasing order, and so on. One can choose any number and two of its neighbors in two opposite directions (horizontal, vertical, or diagonal). Then either the number is increased by 2 and its neighbors are decreased by 1, or the number is decreased by 2 and its neighbors are increased by 1. After several such operations the table again contains all the numbers 1,2,…,100. Prove that they are in the original order.
Great list!
This looks good, but it's too long (and hence scary) in its current form.
I would suggest breaking it up into 5+ different posts / problems, so that the bite-sized chunks are more easily digestible over a period of time, and your audience would not be put off by the length of it.
You could call them Invariants 1, Invariants 2, etc, so that others who enter halfway can still find your other posts.
After you have published the entire series, I would be interested in converting what you have into a post in the Techniques Trainer :)
Ok the diagrams might have some problems. I'll fix them as soon as I have time.
Well, recently at my SIMO training we did combi, so I read up a bit more and tried some imo problems. So I can probably write a while or then about combi.
EDIT: In fact, what are we gonna focus on? Olympiad or general math? Or both?
EDIT again: Yup I went to read, so can I write more primarily on Olympiad? :D
Write what you are interested in.
There is less of a distinction between Olympiad and 'General', and it mainly arises from the exposure of the individual. For example, to someone learning about combinations, the Catalan numbers could be an amazing unexplainable fact. However, by the time you get to statistics in University, the reflection principle would be just another basic fact.
Looking at the main body of this discussion, I guess both. :)
Edit: I guess that's your decision. ;)
Anqi,
This the Titu Lemma part you wanted :
Titu's lemma
The Inequality
Let x1,x2,...,xn be real numbers and y1,y2,...,yn be positive real numbers.
Then, the following inequality holds,
y1x12+y2x22+...+ynxn2≥y1+y2+...+yn(x1+x2+...+xn)2
Why is this true? Actually, this inequality follows from the Cauchy-Schwarz Inequality.
(y1x12+y2x22+...+ynxn2)(y1+y2+...+yn)≥(x1+x2+...+xn)2
Examples
1. Prove Nesbitt Inequality :
b+ca+a+cb+b+ac≥23
Solution :
We can't see any squares terms on the numerators, so wishful thinking motivates us to create them.
How can we create the square terms? Squaring the whole left hand side is very messy, and a much simpler way is to multiply the numerators and denominators of the fractions by a,b,c respectively.
Now, the way to proceed is clear, as now by Titu's Lemma we get
ab+aca2+ab+bcb2+bc+acc2
≥2(ab+bc+ca)(a+b+c)2
Now, we just have to prove that 2(a+b+c)2≥6(ab+bc+ca), which can be rewritten as
(a+b+c)2≥3(ab+bc+ca), which can be rewritten as a2+b2+c2≥ab+bc+ca,
The last inequality follows from 21[(a−b)2+(b−c)2+(a−c)2]≥0.
2. (TOT 1998) Prove that for any positive real numbers a,b,c,
a2+ab+b2a3+b2+bc+c2b3+a2+ac+c2c3≥3a+b+c
Solution :
Again, we want to make the numerators on the left hand side be squares.
So, we again multiply the numerators and denominators of the fractions by a,b,c respectively.
By Titu's Lemma,
a(a2+ab+b2)a4+b(b2+bc+c2)b3+c(a2+ac+c2)c3
≥a(a2+ab+b2)+b(b2+bc+c2)+c(a2+ac+c2)(a2+b2+c2)2
But the denominator is equivalent to (a+b+c)(a2+b2+c2).
Thus, we the inequality becomes
a(a2+ab+b2)a4+b(b2+bc+c2)b3+c(a2+ac+c2)c3≥a+b+ca2+b2+c2
Finally, we have to prove that a+b+ca2+b2+c2≥3a+b+c, which is true. (The proof is similar to the last problem.))
3. (IMO 1995) Let a,b,c be positive real numbers such that abc=1. Prove that
a3(b+c)1+b3(a+c)1+c3(b+a)1≥23.
Solution : Direct application of Titu's Lemma is unfortunately doomed to failure. We divide the numerator and denominator by a2,b2 and c2 respectively.
Now, we have by Titu's Lemma,
a3(b+c)1+b3(a+c)1+c3(b+a)1=a(b+c)a21+b(a+c)b21+c(b+a)c21
≥2(ab+bc+ca)(a1+b1+c1)2=2(ab+bc+ca)(ab+bc+ca)2
≥2ab+bc+ca≥233(abc)2=23
where the last inequality holds by AM-GM.
This proves the inequality.
Problems
1. Prove that for all positive real numbers x,y,z
x+y2+y+z2+z+x2≥x+y+z9
2. Prove that for all positive real numbers a,b,c,d
b+2c+3da+c+2d+3ab+d+2a+3bc+a+2b+3cd≥32
3. Problem link
4. (IMO 2005)
Prove that for all positive real numbers x,y,z such that xyz≥1, then
x5+y2+z2x2+y2+z2+y5+x2+z2x2+y2+z2+z5+y2+x2x2+y2+z2≤3
To add, here will be the "normal" Cauchy Schwarz: (I will be adding more stuff)
Cauchy Schwarz Inequality: Let (a1,a2,…,an) and (b1,b2,…,bn) be two sequences of real numbers, then we have:
(a12+a22+…+an2)(b12+b22+…+bn2)≥(a1b1+a2b2+…+anbn)2
In particular, equality holds iff there exists k∈R for which ai=kbi for i=1,…,n.
Proof: We will present 2 proofs, one originating from analysis on the equality case, the other by wishful thinking on small cases of n=2,3.
(i) Consider defining the following function f:
f(x)=(a1x−b1)2+(a2x−b2)2+…+(anx−bn)2.
We will expand this to get:
f(x)=(a12+a22+…+an2)x2−2(a1b1+a2b2+…anbn)x+(b12+b22+…+bn2)
From our first way of representing f(x), we can conclude that f(j)≥0↔∆f≤0 or
(a12+a22+…+an2)(b12+b22+…+bn2)≥(a1b1+a2b2+…+anbn)2
Equality holds if the equation f(x)=0 has one root.■
(ii) Just remark that:
(a12+a22+…+an2)(b12+b22+…+bn2)−(a1b1+a2b2+…+anbn)2=∑i,j=1n(aibj+ajbi)2≥0■
Let us note the following positive things regarding Cauchy-Schwarz:
it is effective in proving symmetric inequalities
Try to form squares
Helps to clear up square roots
Example 1: (Iran 1998) Suppose that x,y,z≥1 and x1+y1+z1=2. Prove that:
x+y+z≥x−1+y−1+z−1.
Solution: The condition given is considerably weird. In a sense, we would like to involve the variables more than just on the denominators. We have 2 approaches, either use a substitution or try to rewrite it in a nicer form. It seems that the latter works. We notice that most conditions have the constant given as 1 so it is pretty motivated to write the hypothesis as:
xx−1+yy−1+zz−1=1.
Now notice how "magically" we have (x−1),(y−1),… on the numerator. In fact, due to the fact that we can square both sides, we can immediately see how to apply Cauchy:
∑cycx=(∑cycx)(∑cycxx−1)≥(∑cycx−1)2. ■
Example 2: (Japan 2004) Let a,b,c be positive real numbers with sum 1. Prove that
1−a1+a+1−b1+b+1−c1+c≤b2a+c2b+a2c.
Solution: With a little experience in inequalities, we see some use for a+b+c=1:
Substitute into the question
Substitue as a=x+y+zx,b=x+y+zy,etc
Now, also note that we do NOT want to make the inequality in question more complicated, so we opt to using the former. Also with some experience we see that we do not like fractions such as 1−a1+a or 1+a1−a for the matter. We like to only have the variable and no constants (or vice versa) on the denominator so that we can easily form squares. Therefore, by substituting and dividing both sides by 2 to make the RHS ba,…, we rewrite the inequality as:
23+b+ca+a+bb+a+bc≤ba+cb+ac
We isolate the constant and rearrange to get:
∑cycb(b+c)ac≥23
Intuitively, we multiply both numerator and denominator of the LHS by ac (taken cyclically), and directly apply Cauchy Schwarz,
∑cycb(b+c)ac=∑cycabc(a+c)a2c2≥2abc(a+b+c)(ab+bc+ca)2≥23■
Example 3: (IMO SL 2011) Given arbitrary real numbers x1,x2,…,xn, prove that:
1+x12x1+1+x12+x22x2+…+1+x12+x22+…+xn2xn.
Solution: We recall the extremely useful variant of Cauchy:
n(a12+a22+…+an2)≥(a1+a2+…+an)2
So, what we do is that, we apply the above inequality directly to the given inequality in question. The motivation behind this step is to generate an n term. So,
(1+x12x1+1+x12+x22x2+…+1+x12+x22+…+xn2xn)2≤[(1+x12x1)2+…+(1+x12+x22+…+xn2xn)2]×n.
Notice also that for k≥2, we have:
(1+x12+x22+…+xk2xk)2=1+x12+x22+…+xk2)2xk2
≤(1+x12+x22+…+xk2)(1+x12+x22+…+xk−12)xk2
≤1+x12+x22+…+xk−121−1+x12+x22+…+xk21.
The idea here is that in order to generate n we might as well try to eliminate the variables by telescoping. Observe finally, that for k=1, we have:
(1+x12x1)2≤1+x12x12=1−1+x121
I leave my reader to finish the argument.■
Anqi, are you satisfied?
Yup nice :) Thanks!
Note also that we can prove Nesbitt's in another way. Essentially we generalise to get the following:
Given positive reals a1,a2,a3,…,an, denote S=a1+a2+…+an, then prove that:
∑k=1nn−akak≥n−1n.
I remember seeing this somewhere, but I forgot the exact source. Eitherways, it can be seen that n=3 corresponds to Nesbitt's.
Proof: Let us rewrite:
(n−1)S=nS−S=nS−(a1+a2+…+an)=(S−a1)+(S−a2)+…+(S−an) and by noticing the trivial n2=(1+1+…+1)2 where there are n 1s, then by Cauchy:
∑k=1n(S−ak)∑k=1nS−ak1≥n2
After substituting in our observation and rearranging,
∑k=1n(1+S−akak)≥n−1n2.
We are done. ■
Problem 2 is from IMO 1993. Here is another problem that I came up that has a strong correlation with that question:
Problem: Given a positive integer n≥3, choose arbitrary a1,a2,…,an, find the minimum value of:
∑i=1nai+2xi+2+…+(n−1)xi+n−1ai
where indices are taken (modn).
We can see that the IMO 1993 problem is just n=4.
I'll give it a try:
Question: We know that i=1∑ni=21n(n+1), i=1∑ni2=61n(n+1)(2n+1), i=1∑ni3=41n2(n+1)2. We can find i=1∑nik, for k=4,5,…. But we need to know the previous sums of power. That is, for example, if we want to find the closed form of i=1∑ni7, we must first know i=1∑nij for j=1,2,3,4,5,6. Is there an alternative way to determine a sum of certain power without knowing their preceding sums?
Question 2: If the divisors of natural number n are a1,a2,a3,…,aj arranged in ascending order, then why is it (usually) the median of these numbers is close to n?
This follows trivially if the number is a square (and thus has an odd number off divisors). If the number is not a square then it has an even number of divisors half of which are more than its square root and half of which are less than it. If we want to find the median we must add the 2 numbers (say aandb) which are just before and after the square root of the number and divide by 2. Now we use the AM-GM inequality (or equality in this case) for 2 numbers. As the 2 numbers approach each other in value the AM and GM come close together. aandb are more close to each other than any of the other divisors of n. We also have [a∗b]1/2=n1/2. Hence [a+b]/2 is close to n1/2
I can't figure out why the editing is coming out like this ><
Try:
a \space \text{and} \space b
Question 3: It's been proven that for any configurations for a Rubik's Cube, the least number of moves to solve it is 20. Let Gn denote the most number of moves for any configuration for a n×n×n cube (so G3=20). How do we evaluate G2,G4,G5,G6,…?
But the article says that with optimal play the MAX number of moves should be 20...
I didn't see that... FIXED.
Question 4: Mathematicians are always interested in the finding the largest prime number, the current record is a Mersenne Prime. But it appears that there are only few primes p such that Mp=2p−1 is also a prime. Don't you think there's a better way of finding prime numbers? Like applying Mill's constant, because it is GUARANTEED a prime number?
Prove Riemann Hypothesis. :D
Alright, give me 15748 years.
Question 5: There's plenty of ways to prove that a harmonic series diverge. Do you have your own proof that's not listed here?
Question 6: We denote SOD(n) as the sum of digits of positive integer n. Is there a formula to determine i=1∑nSOD(i) ? What if the integer is written in other base representation (i.e: binary, hexadecimal)?
Posts will happen from an individuals account, with the tag of "Torque Group". This way, others will be able to identify who the originator is, and also find similar interesting material through the tags.
As such, you do not need to choose 1 area to focus on. In fact, having a variety of posts would be preferable, so that you get a broad coverage. I anticipate that different people will focus on specific topics that interest them, and there isn't a requirement for cohesion as a group.
It would be good to think about how you want to structure the timing of the posts. Having 1-2 posts a day would be great, but would of course depend on your availability as a group. I would also expect that other members would be interested in contributing similar posts, though at a lower frequency.
Hmm if you refer to my draft post below, then you will see that it contains quite a lot of material - in fact, it does contain a lot of examples from varying sources such as IMO and my training notes. As such, since I'm still terrible at LaTeX, it takes me quite some time to type this out. So probably I can do up to 2 posts per week until end of december and when my new school term starts, probably 1 post per week. How about the other members?
EDIT: I mean my draft post above.
2 posts per week is perfect. I meant 1-2 posts a day as a group, and so with 10 members it averages out to 1+ post per week per person.
I came across this interesting problem :
There are 30 people in a party. Each of them has exactly one favorite chess variant and exactly one favorite classical inequality. Each person lists this information in a survey. Among the survey responses, there are exactly 20 different favorite chess variant and exactly 10 different favorite inequalities. Let n be the number of people M such that the number of people who listed M's favorite inequality is greater than the number of people who listed M's favorite chess variant. Prove that n≥11.
Nice problem! Because I am quite bored, let me give it a try.
Let the 20 different chess variant be c1,…,c20 and also let the 10 different classical inequalities be e1,…,e10 (I chose e instead of i to avoid ii ). We will define, intuitively, the sets Pi (ok, note that clearly 1≤i≤20 ) that chose ci as their favourite chess variant and Qi (again, obviously 1≤i≤10 ) that chose ei as their favourite classical inequality.
Well, the only way that I can think to proceed with 2 variables and we want to associate the things together, is to sort of try a coordinate approach. So we will assign a value (Xk,Yk) to each party-goer k. So let k∈Pi, then we will let Xk=∣Pi∣1 (where absolute value denotes the cardinality as usual), similarly, if k∈Qj then we will let Yk=∣Qj∣1. Well, the beauty of taking coordinates now reduce the problem to finding party-goers k such that Xk>Yk. And talking about motivation, we did not take Xk=∣Pi∣ because we want to use all the conditions in the problem, we should try to sum up the coordinates, and taking reciprocals make it more predictable.
So now, consider the sum of the coordinates of all the party-goers. The x coordinates sum to 20 and the y coordinates sum to 10. So:
∑kXk−Yk=10.
At this point I thought the answer was 10, but that was seriously naive. Because I forgot about the reciprocals, so if we recall the fractions, we in fact have: Xk−Yk<Xk≤1. So at least 11 terms in the summation needs to be positive, or in the question, n≥11.
EDIT: Btw, where did u get this nice problem? And the people at the party must all be very mathy!
This is from the book "102 Problems In Combinatorics".
By the way, your solution is almost the same as the solution in the book.
Problem 2 : A finite set of (distinct) positive integers is called a DS-set if each of the integers divides the sum of all the numbers in the set. Prove that every finite set of positive integers is a subset of some DS-set.
Sorry, I was bored again, so I'll attempt this problem too. (Just to see whether I improved in combi/number theory)
Consider rephrasing this problem in set notation for easier reference:
Restatement of problem: We are given a finite set of positive integers, which we will denote as X. Prove that we can always find a superset of X, let's call it Y (or in set notation Y⊇X ), that satisfies the conditions: every element of Y divides the sum of the elements in Y.
Normally, when we consider combinatorics problem regarding sets, we can always try to sort the elements, or in our case, we shall use the extremal principle. Consider the maximum element in X, let's denote it as n. We will construct a set T that contains 1…n. EDIT: Right, I forgot to mention that the most key idea of all is to realise the trivial lcm(1,…,n)=n! so that we try to make the sum n!. The motivation behind this construction is that 1+2+…+n=2n(n+1) and then try to telescope the others, and the other main idea is to work backwards. In a sense by our key method of telescoping, the last term should be something like n×(n−1)×(n−2)×…×3×1 we can then work backwards to find the previous term, etc.
In a nutshell, it should be clear that for n≥6, we should get the following sequence for Y:
1,2,…,n,2n(n−3),n(n−1)(n−3),n(n−1)(n−2)(n−4),…,n(n−1)…(3)1
By construction, these sum to n!, and all the elements are distinct and divide n!.
However, for the remaining case n<6 by the sole definition of superset we can see that we just take the construction of n=6. □
EDIT: Perhaps a more "direct" approach is using induction with the following lemma in mind:
Lemma: Given arbitrary x,y we can construct a set satisfying the conditions listed in the problem that contains x,y as its smallest element.
The motivation for considering smallest element is because we add in larger elements, etc.
Problem 6 :
Let a,b,c be positive real numbers with product 1. Prove that
a+b+11+b+c+11+c+a+11≤1.
This looks really symmetric, so there should be no harm in clearing the denominators. In fact, we get something quite neat:
Σcyca2(b+c)+2abc≥2+2(a+b+c)
so now, in this reduced form, we see that we need to bound the sum and product of a,b,c, one of which is given. So, subbing in 2abc≥2 and ∑cyca≥3:
∑cyca2(b+c)≥2∑cyca
Ok.. this looks terrible in its current form, let me expand it:
a2b+a2c+b2a+b2c+c2a+c2b≥2(a+b+c)
Suppose we directly apply AM-GM (by spamming on a2(b+c) is bad because then we get a bound in the opposite direction):
∑(a2b+a2c)≥∑2a4bc≥∑2a3 (here ∑ denotes cyclic sum)
which sucks since we can't square root a third power. So darn. But we can easily patch up by introducing a new variable that does not affect the product - the mightiest 1! So, this time, we have:
∑(a2b+a2c+1)−3≥∑33a4bc−3
≥∑33a3−3=3(a+b+c)−3=2(a+b+c)+(a+b+c−3)
≥2(a+b+c)
as desired. □
Note: this can be simplified at least 10 times but with motivation, the solution becomes more instructive. I wonder if a substitution such as a=yx and it's cyclic counterparts will work.
There is a more elegant way. :)
(a3−b3)(a2−b2)≥0, right?
The idea for that A1, if I am not wrong, is to use the fact thatWell, the first step is to let abc=c3 (ok c≥1 by definition) so that we can in fact sub in a=c3x and cyclically for the rest. Clearly, xyz=1 (this is one of reasons to do such a substitution → to force out more conditions). Just sub everything in and a few manipulation would give:
∑1+a+b1=∑1+k(x3+y3)1
≤1+(x3+y3)1 (the beauty of such a substitution is that we can reduce the number of variables and give ourselves more conditions to work with)
≤∑xyz+x2y+xy21=∑xy1×x+y+z1=1
as desired. □
EDIT: Oops, instead of using the condition xyz=1 I used xyz≥1 in my workings. Actually, it is the same thing, just ignore some parts of the definition.
Problem 8 :
The positive integers from 1000 to 2999 (inclusive) are written on a board.
Each second, one is allowed to erase two numbers a and b on the board, and replace them by 2min(a,b).
After 1999 seconds, one obtains a number c on the board. Prove that c<1.
Question 12: If α,β,γ are the roots to the equation 3x2−1=x2, without combining the three fractions, evaluate 2+α3−α+2+β3−β+2+γ3−γ
Question 13: Prove that cos6∘1+sin24∘1+sin48∘1=sin12∘1
Nice. I wasn't aware of this identity.
Question 14: Find the sixth digit from the end of the number 666666
Question 15: Evaluate x→0limcosx−11+tanx−1+sinx
Problem: Given positive real numbers x,y,z that satisfy x+y+z=1. Prove that
2x2(y+z)x2+yz+2y2(z+x)y2+zx+2z2(x+y)z2+xy≥1.
Here's another inequality from Iran 1998, that I particularly relish.
Problem: Prove that if x,y,z>0 satisfy x1+y1+z1=2, then:
x−1+y−1+z−1≤x+y+z
Hint : Use Cauchy. However, just to give the interested reader a clue, let us apply Cauchy in the most obvious way:
x−1+y−1+z−1≤3(x+y+z−2)
Then the final step would be to prove 3(x+y+z−2)≤x+y+z⇔x+y+z=29. This restated form is not true since we can easily verify that the reverse holds. To find the correct "coefficients" (this is much like matching coefficients in polynomial problems), try the method in Sequential Roots.
Recursion
Recursion is using smaller cases to construct larger cases.
Here are some signs that recursion might be a good method to attack a problem:
The problem asks for the value of a large case, and you can determine small values or they are given.
There is a clear connection between small cases and large cases.
Here are some steps helpful when applying recursion to a problem:
Analyze small cases.
Try to construct larger cases using smaller cases.
Make a conjecture about a connection between a larger case and the case[s] smaller than it.
Prove your conjecture.
Determine the answer using the connection you found.
Let us begin with a simple example:
A collection of 8 cubes consists of one cube with edge-length k for each integer k, 1≤k≤8. A tower is to be built using all 8 cubes according to the rules:
Any cube may be the bottom cube in the tower.
The cube immediately on top of a cube with edge-length k must have edge-length at most k+2.
Let T be the number of different towers than can be constructed. What is the remainder when T is divided by 1000? (AIME)
Your initial instinct may be to try casework based on which position a certain block is in, or maybe you thought that complementary counting was a good approach. However, recursion turns out to be a very fruitful approach to solve this problem. Looking back at our signs:
The problem asks for the value of a large case, and you can determine small values or they are given.
Yes, the problem asks about a tower with 8 cubes, and finding the number of towers of 1, 2 and even 3 is very simple, as the restriction doesn't matter.
There is a clear connection between small cases and large cases.
Yes, to get from 4 cubes to 5 cubes, simply add a cube, although the method of doing that may be slightly trickier.
Let Tn be the number of towers of n cubes. Let us apply our steps above:
Analyze small cases.
With 1 cube, there is clearly only 1 possible tower, so T1=1. With 2 cubes, there are 2 possible towers, and T2=2. With 3 cubes, there are 6 possible towers, and T3=6.
Try to construct larger cases using smaller cases.
If we have 3 cubes, and want to place the 4th, where can we put it? We can clearly put it on the bottom by the first rule in the problem. Following that thought, we can put it on top of the 2 or the 3, but not the 1. Then, we have 3 places we can put the 4th cube, and so T4=T3⋅3=6⋅3=18.
If we have 4 cubes, and want to place the 5th, where can we put it? We can put it on the bottom, or we can put it on top of the 3 or the 4. Therefore, there are 3 places we can put the 5th cube, and T5=T4⋅3=18⋅3=54.
Make a conjecture about a connection between a larger case and the case[s] smaller than it.
We can conjecture that Tk=3Tk−1 for k≥3. Luckily, this connection only depends on the previous term, so the work is easier.
Prove your conjecture.
If k≥3, then the kth cube can go on the bottom, or on top of the (k−1)th cube or the (k−2)th cube. Then, we conclude that Tk=3Tk−1, as for each tower with k−1 cubes, the kth cube can go in 3 places.
Determine the answer using the connection you found.
Finally, T2=2, and T8=3(T7)=32(T6)=⋯=36(T2)=1458.
Well, that wasn't that bad, was it? I guarantee casework and other methods would have taken longer. Now that we have seen a more basic application of recursion, let us dive in deeper with another problem.
In how many ways can 7 teams place in a tournament if ties are allowed?
First, let us try to understand the problem, and I think a good way to go about that is by listing some possible orderings. Let the teams be numbered 1 through 7. We could have 1 be first, 2 be second, and so on till 7 which is last, or we could have 1 and 3 and 6 tie for first, and the other four teams tie for second, or we could... You know what, I'm starting to think that there might be a few too many possibilities to list. In fact, even without ties, there are 7!=5040 possibilities.
Now, let us try casework. If there are no ties, there are 5040 possibilities. lf there is one tie between two teams for first, we first pick the two teams that tie, and then order the remaining five teams. In fact, the number of possible placements, assuming teams are indistinguishable, is equal to the number of ordered partitions of 7, which you will soon see in problem number 1 below. I'll tell you this much: it is way too large to do by hand.
As this post is about recursion, I hope you are now thinking about how to apply recursion to this problem. Sign number 1 (asks for a large case, small cases are determinable) is satisfied. Number 2 (connect larger cases with smaller cases) is also satisfied, as all we do is add a team. Now, let us try to think about how a recursion would work...
If we have 6 teams, and want to place the 7th, where can be put it? If there are no ties in the 6 teams, this question is not so difficult. We can have 7 tie with any of the other 6 teams, or we can have it not tie, and be first, second, third, ..., or last. But what if there are ties? Say there is one tie. Then, we can have 7 tie with any of the 5 (do you see why 5 now?) other places, or it can not tie, being first, second, third, ..., or last. Continuing this train of thought, if we knew how many possibilities there were for 6 teams in 4 "groups," then we could place the 7th team tying with one of the 4 groups, or it could be alone in 5 places. This starts to sound like a recursion we can do!
To generalize, say we have n teams in k groups in x different ways. We can have n+1 teams in k groups by adding a team to any of the k existing groups, giving kx ways, and we can have n+1 teams in k+1 groups by adding a team alone in k+1 different locations, giving (k+1)x ways.
Now, let us start a recursion. Let f(n,k) be the number of ways n teams can place into k spots, and define f(n,0)=0 and f(n,k)=0 if k>n. By our earlier logic, f(n+1,k)=(k)f(n,k−1)+(k)f(n,k)=k(f(n,k−1)+f(n,k)) For the base case, we can see that f(1,1)=1. Let us make a table, where n goes down the side and k is on the top: 12345671f(1,1)f(2,1)f(3,1)f(4,1)f(5,1)f(6,1)f(7,1)20f(2,2)f(3,2)f(4,2)f(5,2)f(6,2)f(7,2)300f(3,3)f(4,3)f(5,3)f(6,3)f(7,3)4000f(4,4)f(5,4)f(6,4)f(7,4)50000f(5,5)f(6,5)f(7,5)600000f(6,6)f(7,6)7000000f(7,7)
To finish, we simply fill in values (don't mess up here!): 1234567111111112026143062126300636150540180640002424015608400500001201800168006000007201512070000005040 and the answer is 1+126+1806+8400+16800+15120+5040=47293.
Problems:
Find the number of ordered partitions of 10 where an ordered partition of n is a tuple of positive integers (a1,a2,⋯,ak) such that a1+a2+⋯+ak=n.
A mail carrier delivers mail to the nineteen houses on the east side of Elm Street. The carrier notices that no two adjacent houses ever get mail on the same day, but that there are never more than two houses in a row that get no mail on the same day. How many different patterns of mail delivery are possible? (AIME)
In how many ways can 7 teams place in a tournament if ties are allowed? Try this problem with a single recursion that doesn't depend on groups. Hint: use more than one previous value in the recursion
See here.
Next week, I will write a post about recursion in computer science.
In two weeks, we will learn how to solve expressions such as Tk=3Tk−1, which are called recurrence relations.
Oh yeah sorry the numbering is not working correctly since I have words in between.
Problem 3 : The sequence given by x0=a,x1=b, and xn+1=21(xn−1+xn1) for all n≥1 is periodic. Prove that ab=1.
Problem 4 :
Suppose that f is a decreasing function from positive reals to positive reals such that for all positive reals x,y,
f(x+y)+f(f(x)+f(y))=f(f(x+f(y))+f(y+f(x))).
Prove that f(f(x))=x.
Problem 5 :
Find all functions f:Q→Q such that
f(x+y)+f(x−y)=2(f(x)+f(y)) for all rational numbers x,y.
Problem 7 :
Find all functions f, defined on the set of ordered pairs of positive integers, satisfying the following properties.
f(x,x)=x,f(x,y)=f(y,x),(x+y)f(x,y)=yf(x,x+y)
Just a note : Our group currently has the most number of comments among the 3 groups.
True that is and it seems that we have been posting quite a lot of problems and ideas compared with some of the other groups :P And also Zi Song, do you have any nice invariant problems that I can include in my invariant post?
Problem 8. (Actually, I created an invariant problem, but I'm going to save that for other uses.)
Well, that's partly because you guys have the most members, and also the most active members. This is why I said I have a lot of confidence in this group and will check in occasionally :)
Random problem I saw: Determine all positive integers n for which there exists a polynomial P(x) with real coefficients, with the following properties:
(1) for each integer k, we have P(k)∈Z↔n∤k;
(2) degf<n.
The key idea here is to notice that if we can somehow find a recursion then we can apply certain coprime arguments, so we think of newtonian interpolation which in a form easy to use, states:
f(j+n)=∑i=0n−1(−1)n−1−i(in)f(j+i).
Remark: This is from the RMM.
EDIT: There are other solutions using the fact that f(0) is not an integer (for obvious reasons).
Problem 9 :
In triangle ABC,
3sinA+4cosB=6,4sinB+3cosA=1
Find angle C.
This is a little bit... too extremely easy. Don't you think?
No worries about the difficulty, you can always cross post into another group :)
Yes, it is. :)
Problem 10 : Find all primes p such that p2+11 has exactly 6 positive divisors.
10 is a nice number, so let me attempt this question.
I see no clear clue on how to proceed, so I shall employ 2 strategies:
Recall Useful Formulas: Remember that if the prime factorisation of an integer n is p1e1p2e2…pnen, then the number of prime factors that it has is given by (e1+1)(e2+1)…(en+1).
Mess around with small values: In this case, let us list out: - (we compute the number of divisors using the formula given above)
p=2. The expression equates to 15 which has 4 divisors, which is bad.
p=3. The expression equates to 20 which indeed has 6 divisors, as we wanted.
p=5. The expression equates to 36 which has 9 divisors.
p=7. The expression equates to 60 which has too many divisors.
These give us information to motivate a conjecture that the only answer is p=3.
Well, clearly in most number theory problems we like to use mods. Intuitively, in most cases, a small mod is used, and I feel particularly inclined to consider 3 and 4 due to the proximity of 11 in question to 12.
Now recall that all primes >3 is odd. We would like to prove that p2+11 for p>3 has lots of factors. (motivated by the observations) However, one can easily verify that an odd square satisfies p2≡1(modi) for i=3,4.
Now, since 11≡−1(mod12), we see that 12∣p2+11 for all primes p>3. But 12 has 6 factors, so any multiple of 12 has >6 factors. We have successfully proven our conjecture. □
Problem 11 : Determine whether it's possible to find a cube and a plane such that the distances from the vertices of the cube to the plane are 0,1,2,...,7.
Question 7: What is the best estimate for positive integer n such that ⌊320⌋+⌊321⌋+⌊322⌋+…+⌊321000⌋≈32n
Question 8: Let a>0 and let P(x) be a polynomial with integer coefficients such that for positive integers n≤50, P(n)=a when n is odd; and P(n)=−a when n is even. What is the last three digits of the smallest value of a?
Question 9: For every integer n≥2, let pow(n) be the largest power of the largest prime that divides n. For example pow(144)=pow(24⋅32)=32. What is the largest integer m such that 2010m divides n=2∏5300pow(n)?
Question 10: Let N be the number of x-intercepts on the graph y=sin(x1) is in the interval (0.0001,0.001). What is N rounded up to the nearest hundred digit?
Question 11: If a polynomial P(x) satisfies P(2x2−1)=2(P(x))2, evaluate (P(2012))7+(P(2013))7−(P(2014))7−(P(2015))7
Problem: Let p, be an odd prime. The sequence (an)n≥0 is defined as follows:
a0=0,a1=1,…,ap−2=p−2
for all n≥p−1, an is the least positive integer that does not form an arithmetic sequence of length p with any of the preceding terms.
Prove that, for all n, an is the number obtained by writing n in base p−1 and reading the result in base p.
Source: USAMO
Hint: Induction.
This is from the book "104 Problems In Number Theory" :) Anqi, do you know where to find the Raffles Invitational Mathematical Olympiad (RIMO) problems?
I came across this randomly when I was flipping through past USAMO problems :P Wait for a while, let me ask my trainer. Will update you as soon as he replies.
Edit : I won't spam your email.
Problem: Suppose that in a certain society, each pair of persons can be classified as either amicable or hostile. We shall say that each member of an amicable pair is a friend of the other, and each member of a hostile pair is a foe of the other. Suppose that the society has n persons and q amicable pairs, and that for every set of three persons, at least one pair is hostile. Prove that there is at least one member of the society whose foes include q(1−4q/n2) or fewer amicable pairs.
Source: USAMO
Hints: Try either:
Using graphs
The "adjacency matrix" method (as we call here in trainings), i.e. consider a table having aij=0 or 1
Given the inequalities questions, can someone do a simple writeup for Cauchy Schwarz?
There are several ways of using it, like Titu's Lemma.
Sure I wouldn't mind. I will write one up after my invariance has been posted.
I'm sorry I haven't been able to comment sooner - I've been studying for my finals. I would love to do a series of posts on game theory, which I plan to research over holiday break. I'm also going to think about some geometry concepts that could be introduced. Regarding the group leader thing, I think that we should rotate people as leaders every month or every couple of months. Do people think that a post on math resources (other websites, books, lectures, youtube channels, etc.) would be fun? I think that we, as the Torque Group, could compile a pretty good one. Your ideas for posts look really great. I can't wait to start publishing things with all of you! Regarding my publishing frequency, I could probably publish something 1-2 times a week.
Oh oops, didn't see this here... I'll get on it :P
though my activity during this month will depend a lot on whether I get into school (hearing friday) -- if yes, then I'll have more free time, if not I'll be slaving over applications. Sorry!
Also by olympiad problems.. what are we talking here. USAMO? IMO? Putnam? Right now I plan on just posting problems I find that I think are cool (and at that level).
Problem: Show that for all primes p,
f(p)=∏k=1p−1k2k−p−1
is an integer.
Note: Do NOT use Legendre's Formula.
Why?
Because, there's a neater way to solve it. Hint: Consider f′(p).
I'm working on creating a list of mathematical resources (books, websites, youtube channels, programs, etc.) Would anyone be interested in helping? If so, please reply with your email address and I'll share the google doc with you.
Yes I'm interested.
EDITED: thanks